ChaseDream

标题: Feifei 79 ---- 一个我怎么也绕不明白的问题。。。 [打印本页]

作者: sunnylbs    时间: 2010-12-11 00:42
标题: Feifei 79 ---- 一个我怎么也绕不明白的问题。。。
Feifei 79:
Despite improvements in treatment for asthma, the death rate from this disease has doubled during the past decade from its previous rate. Two possible explanations for this increase have been offered. First, the recording of deaths due to asthma has become more widespread and accurate in the past decade than it had been previously. Second, there has been an increase in urban pollution. However, since the rate of deaths due to asthma has increased dramatically even in cities with long-standing, comprehensive medical records and with little or no urban pollution, one must instead conclude that the cause of increased deaths is the use of bronchial inhalers by asthma sufferers to relieve their symptoms.
Which one of the following is an assumption on which the argument depends?

A.        Urban pollution has not doubled in the past decade.


B.        Doctors and patients generally ignore the role of allergies in asthma.


C.        Bronchial inhalers are unsafe, even when used according to the recommended instructions.


D.       The use of bronchial inhalers aggravates other diseases that frequently occur among asthma sufferers and that often lead to fatal outcomes even when the asthma itself does not.


E.        Increased urban pollution, improved recording of asthma deaths, and the used of asthma deaths, and the use of bronchial inhalers are the only possible explanations of the increased death rate due to asthma.


问题1: E中“and the used of asthma deaths”怎么解释?并且这个因素在原文中没有提到啊?所以这个选项明明应该是削弱的啊,即不是pollusion, 不是recording, 不是bronchial inhalers,还有the used of asthma deaths....怎么能选E呢。。。


问题2: C错在哪了呢?就是因为说B/I本身也不安全,所以即便不是Assumption,好歹也算一个support啊,怎么费费解释里给的是”可以算无关,也可以算削弱”呢?



作者: sdcar2010    时间: 2010-12-11 01:22
E is the answer.  It should be "the use of bronchial inhalers "  There are only three choices for asthma death, two of which were eliminated. The only one left standing is the use of bronchial inhalers.

C is wrong because it is not an assumption for the conclusion to hold. If you negate it, i.e., bronchial inhalers is safe to use, the conclusion that the use of bronchial inhalers causes asthma death still holds. So answer choice C has no impact on the conclusion of the argument.
作者: dwane1113    时间: 2010-12-11 16:46
个人认为:E中的and the used of asthma deaths不是他因,(A)Increased urban pollution, (B)improved recording of asthma deaths, and the used of asthma deaths, and(C) the use of bronchial inhalers 。这个句子大并列里面包含小并列。第二个原因说的是对于现在和以前(used)的asthma deaths的recording提高了。如果是四个原因,这个题就出现语法错误了啊!
作者: sdcar2010    时间: 2010-12-12 09:54
E should read as:

E.        Increased urban pollution, improved recording of asthma deaths, and the use of bronchial inhalers are the only possible explanations of the increased death rate due to asthma.
作者: sunnylbs    时间: 2010-12-14 00:51
This is great! Thank you so much, everybody~
作者: cooperlee01    时间: 2011-6-27 22:25
bronchial inhalers is safe to use, the conclusion that the use of bronchial inhalers causes asthma death still holds.  如果BI是safe的,BI cause asthma death 就不holds了呀,不是都说safe了吗?
作者: sdcar2010    时间: 2011-6-27 22:44
bronchial inhalers is safe to use, the conclusion that the use of bronchial inhalers causes asthma death still holds.  如果BI是safe的,BI cause asthma death 就不holds了呀,不是都说safe了吗?
-- by 会员 cooperlee01 (2011/6/27 22:25:19)



This is a technique called NEGATION, which is specific for necessary assumptions. Read more about the negation method. Then come back and re-do this problem.
作者: cooperlee01    时间: 2011-6-27 23:08
bronchial inhalers is safe to use, the conclusion that the use of bronchial inhalers causes asthma death still holds.  如果BI是safe的,BI cause asthma death 就不holds了呀,不是都说safe了吗?
-- by 会员 cooperlee01 (2011/6/27 22:25:19)




This is a technique called NEGATION, which is specific for necessary assumptions. Read more about the negation method. Then come back and re-do this problem.
-- by 会员 sdcar2010 (2011/6/27 22:44:14)



如果BI是safe的,原文的结论the cause of increased deaths is the use of bronchial inhalers会被推翻。从题目的结构上看,E更像的答案。可心理还是没法说服自己放弃C。
作者: canallaker    时间: 2011-7-16 21:51
我也选C 也不明白为何不对




欢迎光临 ChaseDream (https://forum.chasedream.com/) Powered by Discuz! X3.3